limits of changing the order of a double integral












0












$begingroup$


I've solved this integral $int_{1}^3 int_{1-y}^{2-y}x^2y,dxdy$ which is 3 1/3. Now I'm trying it with different integral order but can't get the limits right. My try was, $int_{0}^1 int_{1-x}^{2-x}x^2y,dydx$.










share|cite|improve this question









$endgroup$












  • $begingroup$
    And the correct one is $int_{-2}^{1}int_{max{1-x,1}}^{min{2-x,3}}x^2y,dy,dx$ (not sure what you do it for).
    $endgroup$
    – metamorphy
    Dec 22 '18 at 6:39


















0












$begingroup$


I've solved this integral $int_{1}^3 int_{1-y}^{2-y}x^2y,dxdy$ which is 3 1/3. Now I'm trying it with different integral order but can't get the limits right. My try was, $int_{0}^1 int_{1-x}^{2-x}x^2y,dydx$.










share|cite|improve this question









$endgroup$












  • $begingroup$
    And the correct one is $int_{-2}^{1}int_{max{1-x,1}}^{min{2-x,3}}x^2y,dy,dx$ (not sure what you do it for).
    $endgroup$
    – metamorphy
    Dec 22 '18 at 6:39
















0












0








0





$begingroup$


I've solved this integral $int_{1}^3 int_{1-y}^{2-y}x^2y,dxdy$ which is 3 1/3. Now I'm trying it with different integral order but can't get the limits right. My try was, $int_{0}^1 int_{1-x}^{2-x}x^2y,dydx$.










share|cite|improve this question









$endgroup$




I've solved this integral $int_{1}^3 int_{1-y}^{2-y}x^2y,dxdy$ which is 3 1/3. Now I'm trying it with different integral order but can't get the limits right. My try was, $int_{0}^1 int_{1-x}^{2-x}x^2y,dydx$.







integration multiple-integral






share|cite|improve this question













share|cite|improve this question











share|cite|improve this question




share|cite|improve this question










asked Dec 22 '18 at 6:29









LLTLLT

32




32












  • $begingroup$
    And the correct one is $int_{-2}^{1}int_{max{1-x,1}}^{min{2-x,3}}x^2y,dy,dx$ (not sure what you do it for).
    $endgroup$
    – metamorphy
    Dec 22 '18 at 6:39




















  • $begingroup$
    And the correct one is $int_{-2}^{1}int_{max{1-x,1}}^{min{2-x,3}}x^2y,dy,dx$ (not sure what you do it for).
    $endgroup$
    – metamorphy
    Dec 22 '18 at 6:39


















$begingroup$
And the correct one is $int_{-2}^{1}int_{max{1-x,1}}^{min{2-x,3}}x^2y,dy,dx$ (not sure what you do it for).
$endgroup$
– metamorphy
Dec 22 '18 at 6:39






$begingroup$
And the correct one is $int_{-2}^{1}int_{max{1-x,1}}^{min{2-x,3}}x^2y,dy,dx$ (not sure what you do it for).
$endgroup$
– metamorphy
Dec 22 '18 at 6:39












3 Answers
3






active

oldest

votes


















2












$begingroup$

A good drawing always helps for these sorts of questions.



enter image description here



As you can see, the region can be broken up into a triangle between $x=-2$ and $-1$, a pair of lines between $x=-1$ and $0$, and another triangle between $x=0$ and $1$.



So hopefully you can see that



$$int_1^3 int_{1-y}^{2-y} x^2y text{d}x text{d}y = int_{-2}^{-1}int_{1-x}^3 x^2y text{d}y text{d}x + int_{-1}^{0} int_{1-x}^{2-x}x^2y text{d}y text{d}x + int_{0}^{1} int_{1}^{2-x}x^2y text{d}y text{d}x$$






share|cite|improve this answer











$endgroup$













  • $begingroup$
    Cool thx! I drew a graph but forgot the area in the negative area. Now everything’s clear. Cheers.
    $endgroup$
    – LLT
    Dec 22 '18 at 6:57










  • $begingroup$
    Sorry, should it be $$int_1^3 int_{1-y}^{2-y} x^2y text{d}x text{d}y = int_{-2}^{-1}int_{1-x}^3 x^2y text{d}y text{d}x + int_{-1}^{0} int_{1-x}^{2-x}x^2y text{d}y text{d}x + int_{0}^{1} int_{1}^{2-x}x^2y text{d}y text{d}x$$. i.e. the upper limit of dy of the first triangle should be 3?
    $endgroup$
    – LLT
    Dec 23 '18 at 6:39












  • $begingroup$
    Indeed it should, that was silly of me
    $endgroup$
    – Cade Reinberger
    Dec 24 '18 at 8:53



















1












$begingroup$

Hint: Try drawing the figure, bounded by
begin{align}
x=2-y\
x=1-y\
y=1\
y=3
end{align}



And then set the limits with y as a function of x first.






share|cite|improve this answer









$endgroup$





















    1












    $begingroup$

    First of all you need to draw a graph of the region of integration which is a parallelogram.



    You notice that you need three integrals to cover the region if you changed the order of integration.



    The x runs from -2 to 1 and you upper and lower limits for y changes on each sub-intervals of length 1.






    share|cite|improve this answer









    $endgroup$













      Your Answer





      StackExchange.ifUsing("editor", function () {
      return StackExchange.using("mathjaxEditing", function () {
      StackExchange.MarkdownEditor.creationCallbacks.add(function (editor, postfix) {
      StackExchange.mathjaxEditing.prepareWmdForMathJax(editor, postfix, [["$", "$"], ["\\(","\\)"]]);
      });
      });
      }, "mathjax-editing");

      StackExchange.ready(function() {
      var channelOptions = {
      tags: "".split(" "),
      id: "69"
      };
      initTagRenderer("".split(" "), "".split(" "), channelOptions);

      StackExchange.using("externalEditor", function() {
      // Have to fire editor after snippets, if snippets enabled
      if (StackExchange.settings.snippets.snippetsEnabled) {
      StackExchange.using("snippets", function() {
      createEditor();
      });
      }
      else {
      createEditor();
      }
      });

      function createEditor() {
      StackExchange.prepareEditor({
      heartbeatType: 'answer',
      autoActivateHeartbeat: false,
      convertImagesToLinks: true,
      noModals: true,
      showLowRepImageUploadWarning: true,
      reputationToPostImages: 10,
      bindNavPrevention: true,
      postfix: "",
      imageUploader: {
      brandingHtml: "Powered by u003ca class="icon-imgur-white" href="https://imgur.com/"u003eu003c/au003e",
      contentPolicyHtml: "User contributions licensed under u003ca href="https://creativecommons.org/licenses/by-sa/3.0/"u003ecc by-sa 3.0 with attribution requiredu003c/au003e u003ca href="https://stackoverflow.com/legal/content-policy"u003e(content policy)u003c/au003e",
      allowUrls: true
      },
      noCode: true, onDemand: true,
      discardSelector: ".discard-answer"
      ,immediatelyShowMarkdownHelp:true
      });


      }
      });














      draft saved

      draft discarded


















      StackExchange.ready(
      function () {
      StackExchange.openid.initPostLogin('.new-post-login', 'https%3a%2f%2fmath.stackexchange.com%2fquestions%2f3049167%2flimits-of-changing-the-order-of-a-double-integral%23new-answer', 'question_page');
      }
      );

      Post as a guest















      Required, but never shown

























      3 Answers
      3






      active

      oldest

      votes








      3 Answers
      3






      active

      oldest

      votes









      active

      oldest

      votes






      active

      oldest

      votes









      2












      $begingroup$

      A good drawing always helps for these sorts of questions.



      enter image description here



      As you can see, the region can be broken up into a triangle between $x=-2$ and $-1$, a pair of lines between $x=-1$ and $0$, and another triangle between $x=0$ and $1$.



      So hopefully you can see that



      $$int_1^3 int_{1-y}^{2-y} x^2y text{d}x text{d}y = int_{-2}^{-1}int_{1-x}^3 x^2y text{d}y text{d}x + int_{-1}^{0} int_{1-x}^{2-x}x^2y text{d}y text{d}x + int_{0}^{1} int_{1}^{2-x}x^2y text{d}y text{d}x$$






      share|cite|improve this answer











      $endgroup$













      • $begingroup$
        Cool thx! I drew a graph but forgot the area in the negative area. Now everything’s clear. Cheers.
        $endgroup$
        – LLT
        Dec 22 '18 at 6:57










      • $begingroup$
        Sorry, should it be $$int_1^3 int_{1-y}^{2-y} x^2y text{d}x text{d}y = int_{-2}^{-1}int_{1-x}^3 x^2y text{d}y text{d}x + int_{-1}^{0} int_{1-x}^{2-x}x^2y text{d}y text{d}x + int_{0}^{1} int_{1}^{2-x}x^2y text{d}y text{d}x$$. i.e. the upper limit of dy of the first triangle should be 3?
        $endgroup$
        – LLT
        Dec 23 '18 at 6:39












      • $begingroup$
        Indeed it should, that was silly of me
        $endgroup$
        – Cade Reinberger
        Dec 24 '18 at 8:53
















      2












      $begingroup$

      A good drawing always helps for these sorts of questions.



      enter image description here



      As you can see, the region can be broken up into a triangle between $x=-2$ and $-1$, a pair of lines between $x=-1$ and $0$, and another triangle between $x=0$ and $1$.



      So hopefully you can see that



      $$int_1^3 int_{1-y}^{2-y} x^2y text{d}x text{d}y = int_{-2}^{-1}int_{1-x}^3 x^2y text{d}y text{d}x + int_{-1}^{0} int_{1-x}^{2-x}x^2y text{d}y text{d}x + int_{0}^{1} int_{1}^{2-x}x^2y text{d}y text{d}x$$






      share|cite|improve this answer











      $endgroup$













      • $begingroup$
        Cool thx! I drew a graph but forgot the area in the negative area. Now everything’s clear. Cheers.
        $endgroup$
        – LLT
        Dec 22 '18 at 6:57










      • $begingroup$
        Sorry, should it be $$int_1^3 int_{1-y}^{2-y} x^2y text{d}x text{d}y = int_{-2}^{-1}int_{1-x}^3 x^2y text{d}y text{d}x + int_{-1}^{0} int_{1-x}^{2-x}x^2y text{d}y text{d}x + int_{0}^{1} int_{1}^{2-x}x^2y text{d}y text{d}x$$. i.e. the upper limit of dy of the first triangle should be 3?
        $endgroup$
        – LLT
        Dec 23 '18 at 6:39












      • $begingroup$
        Indeed it should, that was silly of me
        $endgroup$
        – Cade Reinberger
        Dec 24 '18 at 8:53














      2












      2








      2





      $begingroup$

      A good drawing always helps for these sorts of questions.



      enter image description here



      As you can see, the region can be broken up into a triangle between $x=-2$ and $-1$, a pair of lines between $x=-1$ and $0$, and another triangle between $x=0$ and $1$.



      So hopefully you can see that



      $$int_1^3 int_{1-y}^{2-y} x^2y text{d}x text{d}y = int_{-2}^{-1}int_{1-x}^3 x^2y text{d}y text{d}x + int_{-1}^{0} int_{1-x}^{2-x}x^2y text{d}y text{d}x + int_{0}^{1} int_{1}^{2-x}x^2y text{d}y text{d}x$$






      share|cite|improve this answer











      $endgroup$



      A good drawing always helps for these sorts of questions.



      enter image description here



      As you can see, the region can be broken up into a triangle between $x=-2$ and $-1$, a pair of lines between $x=-1$ and $0$, and another triangle between $x=0$ and $1$.



      So hopefully you can see that



      $$int_1^3 int_{1-y}^{2-y} x^2y text{d}x text{d}y = int_{-2}^{-1}int_{1-x}^3 x^2y text{d}y text{d}x + int_{-1}^{0} int_{1-x}^{2-x}x^2y text{d}y text{d}x + int_{0}^{1} int_{1}^{2-x}x^2y text{d}y text{d}x$$







      share|cite|improve this answer














      share|cite|improve this answer



      share|cite|improve this answer








      edited Dec 24 '18 at 8:53

























      answered Dec 22 '18 at 6:41









      Cade ReinbergerCade Reinberger

      38317




      38317












      • $begingroup$
        Cool thx! I drew a graph but forgot the area in the negative area. Now everything’s clear. Cheers.
        $endgroup$
        – LLT
        Dec 22 '18 at 6:57










      • $begingroup$
        Sorry, should it be $$int_1^3 int_{1-y}^{2-y} x^2y text{d}x text{d}y = int_{-2}^{-1}int_{1-x}^3 x^2y text{d}y text{d}x + int_{-1}^{0} int_{1-x}^{2-x}x^2y text{d}y text{d}x + int_{0}^{1} int_{1}^{2-x}x^2y text{d}y text{d}x$$. i.e. the upper limit of dy of the first triangle should be 3?
        $endgroup$
        – LLT
        Dec 23 '18 at 6:39












      • $begingroup$
        Indeed it should, that was silly of me
        $endgroup$
        – Cade Reinberger
        Dec 24 '18 at 8:53


















      • $begingroup$
        Cool thx! I drew a graph but forgot the area in the negative area. Now everything’s clear. Cheers.
        $endgroup$
        – LLT
        Dec 22 '18 at 6:57










      • $begingroup$
        Sorry, should it be $$int_1^3 int_{1-y}^{2-y} x^2y text{d}x text{d}y = int_{-2}^{-1}int_{1-x}^3 x^2y text{d}y text{d}x + int_{-1}^{0} int_{1-x}^{2-x}x^2y text{d}y text{d}x + int_{0}^{1} int_{1}^{2-x}x^2y text{d}y text{d}x$$. i.e. the upper limit of dy of the first triangle should be 3?
        $endgroup$
        – LLT
        Dec 23 '18 at 6:39












      • $begingroup$
        Indeed it should, that was silly of me
        $endgroup$
        – Cade Reinberger
        Dec 24 '18 at 8:53
















      $begingroup$
      Cool thx! I drew a graph but forgot the area in the negative area. Now everything’s clear. Cheers.
      $endgroup$
      – LLT
      Dec 22 '18 at 6:57




      $begingroup$
      Cool thx! I drew a graph but forgot the area in the negative area. Now everything’s clear. Cheers.
      $endgroup$
      – LLT
      Dec 22 '18 at 6:57












      $begingroup$
      Sorry, should it be $$int_1^3 int_{1-y}^{2-y} x^2y text{d}x text{d}y = int_{-2}^{-1}int_{1-x}^3 x^2y text{d}y text{d}x + int_{-1}^{0} int_{1-x}^{2-x}x^2y text{d}y text{d}x + int_{0}^{1} int_{1}^{2-x}x^2y text{d}y text{d}x$$. i.e. the upper limit of dy of the first triangle should be 3?
      $endgroup$
      – LLT
      Dec 23 '18 at 6:39






      $begingroup$
      Sorry, should it be $$int_1^3 int_{1-y}^{2-y} x^2y text{d}x text{d}y = int_{-2}^{-1}int_{1-x}^3 x^2y text{d}y text{d}x + int_{-1}^{0} int_{1-x}^{2-x}x^2y text{d}y text{d}x + int_{0}^{1} int_{1}^{2-x}x^2y text{d}y text{d}x$$. i.e. the upper limit of dy of the first triangle should be 3?
      $endgroup$
      – LLT
      Dec 23 '18 at 6:39














      $begingroup$
      Indeed it should, that was silly of me
      $endgroup$
      – Cade Reinberger
      Dec 24 '18 at 8:53




      $begingroup$
      Indeed it should, that was silly of me
      $endgroup$
      – Cade Reinberger
      Dec 24 '18 at 8:53











      1












      $begingroup$

      Hint: Try drawing the figure, bounded by
      begin{align}
      x=2-y\
      x=1-y\
      y=1\
      y=3
      end{align}



      And then set the limits with y as a function of x first.






      share|cite|improve this answer









      $endgroup$


















        1












        $begingroup$

        Hint: Try drawing the figure, bounded by
        begin{align}
        x=2-y\
        x=1-y\
        y=1\
        y=3
        end{align}



        And then set the limits with y as a function of x first.






        share|cite|improve this answer









        $endgroup$
















          1












          1








          1





          $begingroup$

          Hint: Try drawing the figure, bounded by
          begin{align}
          x=2-y\
          x=1-y\
          y=1\
          y=3
          end{align}



          And then set the limits with y as a function of x first.






          share|cite|improve this answer









          $endgroup$



          Hint: Try drawing the figure, bounded by
          begin{align}
          x=2-y\
          x=1-y\
          y=1\
          y=3
          end{align}



          And then set the limits with y as a function of x first.







          share|cite|improve this answer












          share|cite|improve this answer



          share|cite|improve this answer










          answered Dec 22 '18 at 6:39









          mm-crjmm-crj

          425213




          425213























              1












              $begingroup$

              First of all you need to draw a graph of the region of integration which is a parallelogram.



              You notice that you need three integrals to cover the region if you changed the order of integration.



              The x runs from -2 to 1 and you upper and lower limits for y changes on each sub-intervals of length 1.






              share|cite|improve this answer









              $endgroup$


















                1












                $begingroup$

                First of all you need to draw a graph of the region of integration which is a parallelogram.



                You notice that you need three integrals to cover the region if you changed the order of integration.



                The x runs from -2 to 1 and you upper and lower limits for y changes on each sub-intervals of length 1.






                share|cite|improve this answer









                $endgroup$
















                  1












                  1








                  1





                  $begingroup$

                  First of all you need to draw a graph of the region of integration which is a parallelogram.



                  You notice that you need three integrals to cover the region if you changed the order of integration.



                  The x runs from -2 to 1 and you upper and lower limits for y changes on each sub-intervals of length 1.






                  share|cite|improve this answer









                  $endgroup$



                  First of all you need to draw a graph of the region of integration which is a parallelogram.



                  You notice that you need three integrals to cover the region if you changed the order of integration.



                  The x runs from -2 to 1 and you upper and lower limits for y changes on each sub-intervals of length 1.







                  share|cite|improve this answer












                  share|cite|improve this answer



                  share|cite|improve this answer










                  answered Dec 22 '18 at 6:49









                  Mohammad Riazi-KermaniMohammad Riazi-Kermani

                  41.6k42061




                  41.6k42061






























                      draft saved

                      draft discarded




















































                      Thanks for contributing an answer to Mathematics Stack Exchange!


                      • Please be sure to answer the question. Provide details and share your research!

                      But avoid



                      • Asking for help, clarification, or responding to other answers.

                      • Making statements based on opinion; back them up with references or personal experience.


                      Use MathJax to format equations. MathJax reference.


                      To learn more, see our tips on writing great answers.




                      draft saved


                      draft discarded














                      StackExchange.ready(
                      function () {
                      StackExchange.openid.initPostLogin('.new-post-login', 'https%3a%2f%2fmath.stackexchange.com%2fquestions%2f3049167%2flimits-of-changing-the-order-of-a-double-integral%23new-answer', 'question_page');
                      }
                      );

                      Post as a guest















                      Required, but never shown





















































                      Required, but never shown














                      Required, but never shown












                      Required, but never shown







                      Required, but never shown

































                      Required, but never shown














                      Required, but never shown












                      Required, but never shown







                      Required, but never shown







                      Popular posts from this blog

                      Quarter-circle Tiles

                      build a pushdown automaton that recognizes the reverse language of a given pushdown automaton?

                      Mont Emei